A camera has a list price of
$
459.99
before tax. If the sales tax rate is
7.25
%
,
find the total cost of the camera with sales tax included.

Round your answer to the nearest cent, if necessary.

Answers

Answer 1
the total cost of the camera with sales tax included is $493.36.
Answer 2

In this case, we'll have to carry out several steps to find the solution.

Step 1:

Data:

camera:

list price (before tax) = $459.99

sales tax = 7.25%

Step 2:

percentage:

[tex]sales \ tax = 7.25\% = 7/100 = 0.07[/tex]

[tex]total \ cost = \$459.99 + \$459.99 \times (0.07) = \$459.99 + \$32.1993 = \$492.1893[/tex]

The answer is:

$492.19


Related Questions

in exercises 47 and 48, find an equation for (a) the tangent to the curve at p and (b) the horizontal tangent to the curve at q

Answers

The tangent to the curve at P is y = -x + (2 + π/2) and the horizontal tangent to the curve at Q is y = 2.2653.

The straight line that most closely resembles (or "clings to") a curve at a given location is known as the tangent line to the curve. It might be thought of as the limiting position of straight lines that pass between the specified point and a neighbouring curve point as the second point gets closer to the first.

Slope of a tangent to a curve at a given point is,

dy/dx

so, dy/dx = 4 + cotx - 2cosecx

dy/dx = 0 + ([tex]\frac{-1}{sin^2x}[/tex]) - 2(-cotx cosecx)

dy/dx = 2(cotx.cosecx) - 1/sin²x

At p(π/2, 0)

dy/dx = -1.

slope is -1 so equation of tangent is given by

y = mx + c

y = (-1)x + c  atp(π/2, 0)

c = 2 + π/2

So y = -x + (2 + π/2) tangent at P.

Tangent at Q is parallel to x-axis

Q (1, y) hence, its shape is O

put the point in curve Q

y = 4 + cot(1) - 2cosec(1)

y = 2.2653

So y = mx+c

y = c

Sp y = 2.2653 is horizontal tangent at point Q.

Learn more about Tangent to the curve:

https://brainly.com/question/28199103

#SPJ4

Experimental and theoretical probability

Answers

(a) Experimental probability (5 or 8) = 0.193

(b) Theoretical probability (5 or 8) = 0.200

(c) As the number of trials increases, we expect the experimental and theoretical probabilities to become closer, though they might not be equal.

How to compare the experimental probability and theoretical probability?

Probability is the likelihood of a desired event happening.

Experimental probability is a probability that relies mainly on a series of experiments.

Theoretical probability is the theory behind probability. To find the probability of an event, an experiment is not required. Instead, we should know about the situation to find the probability of an event occurring.

(a) From these results, the experimental probability of getting a 5 or 8 will be:

Experimental probability (5 or 8) = P(5) + P(8)

Experimental probability (5 or 8) = (15/150) + (14/150)

Experimental probability (5 or 8) = 29/150

Experimental probability (5 or 8) = 0.193

(b) The theoretical probability of getting a 5 or 8 will be:

0,1, 2, 3, 4, 5, 6, 7, 8, 9

Theoretical probability (5 or 8) = P(5) + P(8)

Theoretical probability (5 or 8) = (1/10) + (1/10)

Theoretical probability (5 or 8) = 2/10

Theoretical probability (5 or 8) = 0.200

(c) As the number of trials increases, we expect the experimental and theoretical probabilities to become closer, though they might not be equal.

Learn more about probability on:

brainly.com/question/251701

#SPJ1

2. problem 4.3.4 for a constant parameter , a rayleigh random variable x has pdf what is the cdf of x?

Answers

The cumulative distribution function (CDF) for given random variable fx(x)  is given by F(x) = 1 - e^[(-a²)(x²/2)]        x > 0,

                           F(x) = 0                                x ≤ 0.

The cumulative distribution function (CDF) F(x) for a Rayleigh random variable X is defined as,

F(x) = P(X ≤ x)

To find the CDF of X, we integrate the PDF of X over the interval [0, x],

F(x) = ∫₀ˣ a²x e^[(-a²)(x²/2)] dx

Using the substitution u = (-a²x²/2),

Simplify the integral as follows,

F(x) = ∫₀ˣ a²x e^[(-a²)(x²/2)] dx

= ∫₀^((-a²x²)/2) -e^u du (where u = (-a²x²/2) and x = √(2u/a²))

= [e^u]₀^((-a²x²)/2)

= 1 - e^[(-a²)(x²/2)]

Therefore, the CDF of X  for the Rayleigh random variable X has PDF fx (x)   is equal to,

F(x) = 1 - e^[(-a²)(x²/2)]        x > 0,

F(x) = 0                                x ≤ 0.

Learn more about random variable here

brainly.com/question/29905758

#SPJ4

The above question is incomplete, the complete question is:

For a constant parameter a > 0, a Rayleigh random variable X has PDF

fx (x)        =    a²xe^[(-a²)(x²/2)]      x > 0

                     0                              otherwise.

What is the CDF of X?

Find the point on the graph of y=x^2+1 that’s closest to the point 8, 1.5. Hint: Remember
the distance formula.

Answers

Answer:

The point on the graph that is closest to the point (8, 1.5) is:

[tex]\left(\sqrt[3]{4}, 2 \sqrt[3]{2}+1\right) \approx \left(1.587,3.520)[/tex]

Step-by-step explanation:

To find the point on the graph of y = x² + 1 that is closest to the point (8, 1.5), we need to find the point on the parabola that is at the shortest distance from (8, 1.5). We can use the distance formula to do this.

[tex]\boxed{\begin{minipage}{7.4 cm}\underline{Distance Formula}\\\\$d=\sqrt{(x_2-x_1)^2+(y_2-y_1)^2}$\\\\\\where:\\ \phantom{ww}$\bullet$ $d$ is the distance between two points. \\\phantom{ww}$\bullet$ $(x_1,y_1)$ and $(x_2,y_2)$ are the two points.\\\end{minipage}}[/tex]

Any point (x, y) on the parabola y = x² + 1 can be defined as (x, x²+1).

Therefore:

(x₁, y₁) = (8, 1.5)(x₂, y₂) = (x, x²+1)

Substitute these points into the distance formula to create an equation for the distance between any point on the parabola and (8, 1.5):

[tex]d = \sqrt{(x - 8)^2 + (x^2+1 - 1.5)^2}[/tex]

Simplifying this expression for d², we get:

[tex]d = \sqrt{(x - 8)^2 + (x^2-0.5)^2}[/tex]

[tex]d^2 = (x - 8)^2 + (x^2-0.5)^2[/tex]

[tex]d^2 = x^2-16x+64 + x^4-x^2+0.25[/tex]

[tex]d^2=x^4-16x+64.25[/tex]

To find the x-coordinate that will minimize this distance, take the derivative of the expression with respect to x, set it equal to zero and solve for x:

[tex]\implies 2d \dfrac{\text{d}d}{\text{d}{x}}=4x^3-16[/tex]

[tex]\implies \dfrac{\text{d}d}{\text{d}{x}}=\dfrac{4x^3-16}{2d}[/tex]

Set it equal to zero and solve for x:

[tex]\implies \dfrac{4x^3-16}{2d}=0[/tex]

[tex]\implies 4x^3-16=0[/tex]

[tex]\implies 4x^3=16[/tex]

[tex]\implies x^3=4[/tex]

[tex]\implies x=\sqrt[3]{4}[/tex]

Finally, to find the y-coordinate of the point on the graph that is closest to the point (8, 1.5), substitute the found value of x into the equation of the parabola:

[tex]\implies y=\left(\sqrt[3]{4}\right)^2+1[/tex]

[tex]\implies y=\sqrt[3]{4^2}+1[/tex]

[tex]\implies y=\sqrt[3]{16}+1[/tex]

[tex]\implies y=\sqrt[3]{2^3 \cdot 2}+1[/tex]

[tex]\implies y=\sqrt[3]{2^3} \sqrt[3]{2}+1[/tex]

[tex]\implies y=2 \sqrt[3]{2}+1[/tex]

Therefore, the point on the graph that is closest to the point (8, 1.5) is:

[tex]\left(\sqrt[3]{4}, 2 \sqrt[3]{2}+1\right) \approx \left(1.587,3.520)[/tex]

Additional information

To find the minimum distance between the point on the graph and (8, 1.5), substitute x = ∛4 into the distance equation:

[tex]\implies d = \sqrt{(\sqrt[3]{4} - 8)^2 + ((\sqrt[3]{4})^2-0.5)^2}[/tex]

[tex]\implies d = 6.72318283...[/tex]

A total of 803 tickets were sold for the school play. They were either adult tickets or student tickets. There were 53 more student tickets sold than adult tickets How many adult tickets were sold? adult tickets *​

Answers

Answer:

375

Step-by-step explanation:

Based on the given conditions, formulate: 53 +2x = 803

Rearrange variables to the left side of the equation:

2x = 803 - 53

Calculate the sum or difference:

2x = 750

Divide both sides of the equation by the coefficient of variable:

x = 750/2

Cross out the common factor: x = 375

I will mark you brainiest!

If the triangles above are reflections of each other, then ∠D ≅ to:
A) ∠F.
B) ∠E.
C) ∠C.
D) ∠A.
E) ∠B.

Answers

Answer:

D I believe

Step-by-step explanation:

Which operation do you use to simplify a ratio after finding the greatest common factor (GCF)?
division
addition
multiplication
subtraction

Answers

Answer:

hey baby

Step-by-step explanation:

hi thwrw honey i love you lol

The operation we use to simplify a ratio after finding the greatest common factor (GCF) is division.

Option A is the correct answer.

What is an expression?

An expression contains one or more terms with addition, subtraction, multiplication, and division.

We always combine the like terms in an expression when we simplify.

We also keep all the like terms on one side of the expression if we are dealing with two sides of an expression.

Example:

1 + 3x + 4y = 7 is an expression.

3 + 4 is an expression.

2 x 4 + 6 x 7 – 9 is an expression.

33 + 77 – 88 is an expression.

We have,

To simplify a ratio after finding the greatest common factor (GCF), we use division.

We divide both terms of the ratio by the GCF.

This reduces the ratio to its simplest form.

Thus,

The operation we use to simplify a ratio after finding the greatest common factor (GCF) is division.

Learn more about expressions here:

https://brainly.com/question/3118662

#SPJ2

A small deck of four cards consists of one red card and three green cards. Draw 7 times with replacement. Assume each draw is a random selection of one card.



Let X = the number of red cards drawn



compute the variance of X. Round to 2 decimal places.



Var(X) =

Answers

The answer of the given question based on probability to compute the variance of X. Round to 2 decimal places the answer is ,Rounding to 2 decimal places, the variance of X is 1.31.

What is Variance?

In statistics, variance is  measure of how spread out or dispersed set of data is. It is calculated as  average of the squared differences from the mean of data. The variance is expressed in units that are square of the units of data, and small variance indicates that data points tend to be close to mean, while a large variance indicates that  data points are spread out over  wider range of values.

To calculate  variance of  set of data, first find mean (average) of the data points. Then, for each data point, subtract  mean from that data point and square the difference. Next, sum up all  squared differences and divide by the total number of data points minus one.

The probability of drawing a red card on any one draw is 1/4, and the probability of drawing a green card is 3/4. Since the draws are made with replacement, the draws are independent, and we can use the binomial distribution to model the number of red cards drawn in 7 draws.

The probability mass function of  binomial distribution with parameters n and p are below:

P(X = k) =(n choose k) *p^k*(1-p)^(n-k)

In this case, we have n = 7 and p = 1/4, so the probability mass function of X is:

P(X = k) = (7 choose k) * (1/4)^k * (3/4)^(7-k)

We can use this formula to calculate the probabilities of X taking each possible value from 0 to 7:

P(X = 0) = (7 choose 0) * (1/4)^⁰ * (3/4)^⁷ ≈ 0.1335

P(X = 1) = (7 choose 1) * (1/4)¹ * (3/4)⁶ ≈ 0.3348

P(X = 2) = (7 choose 2) * (1/4)² * (3/4)⁵ ≈ 0.3119

P(X = 3) = (7 choose 3) * (1/4)³ * (3/4)⁴ ≈ 0.1451

P(X = 4) = (7 choose 4) * (1/4)⁴ * (3/4)³ ≈ 0.0415

P(X = 5) = (7 choose 5) * (1/4)⁵ * (3/4)² ≈ 0.0064

P(X = 6) = (7 choose 6) * (1/4)⁶ * (3/4)¹ ≈ 0.0005

P(X = 7) = (7 choose 7) * (1/4)⁷ * (3/4)⁰ ≈ 0.0000

To calculate the variance of X, we need to calculate the expected value of X and the expected value of X squared:

E(X) = Σ k P(X = k) = 0P(X=0) + 1P(X=1) + 2P(X=2) + 3P(X=3) + 4P(X=4) + 5P(X=5) + 6P(X=6) + 7P(X=7) ≈ 1.75

E(X^2) = Σ k²P(X = k) = 0²P(X=0) + 1²P(X=1) + 2²P(X=2) + 3²P(X=3) + 4²P(X=4) + 5²P(X=5) + 6²P(X=6) + 7²P(X=7) ≈ 4.56

Then, we can use the formula for the variance:

Var(X) = E(X²) - [E(X)]² ≈ 4.56 - (1.75)² ≈ 1.03

Rounding to 2 decimal places, the variance of X is 1.31.

To know more about  Probability mass function visit:

https://brainly.com/question/30765833

#SPJ1

Assuming each draw is a random selection of one card and X = number of red cards drawn. So, the  variance of X rounded to two decimal places is 1.31.

What is Variance?

In statistics, variance is  measure of how spread out or dispersed set of data is. It is calculated as  average of the squared differences from the mean of data. The variance is expressed in units that are square of the units of data, and small variance indicates that data points tend to be close to mean, while a large variance indicates that  data points are spread out over  wider range of values.

The probability of drawing a red card on any one draw is 1/4, and the probability of drawing a green card is 3/4. Since the draws are made with replacement, the draws are independent, and we can use the binomial distribution to model the number of red cards drawn in 7 draws.

The probability mass function of  binomial distribution with parameters n and p are below:

P(X = k) =(n choose k) [tex]p^{k}*(1-p)^{n-k}[/tex]

In this case,

we have n = 7 and p = 1/4, so the probability mass function of X is:

P(X = k) = (7 choose k) * [tex](1/4)^{k}*(3/4)^{7-k}[/tex]

We can use this formula to calculate the probabilities of X taking each possible value from 0 to 7:

P(X = 0) = (7 choose 0) × (1/4)⁰ × (3/4)⁷

≈ 0.1335

P(X = 1) = (7 choose 1) × (1/4)¹ × (3/4)⁶

≈ 0.3348

P(X = 2) = (7 choose 2) × (1/4)² × (3/4)⁵

≈ 0.3119

P(X = 3) = (7 choose 3) × (1/4)³ × (3/4)⁴

≈ 0.1451

P(X = 4) = (7 choose 4) × (1/4)⁴ × (3/4)³

≈ 0.0415

P(X = 5) = (7 choose 5) × (1/4)⁵ × (3/4)²

≈ 0.0064

P(X = 6) = (7 choose 6) × (1/4)⁶ × (3/4)¹

≈ 0.0005

P(X = 7) = (7 choose 7) × (1/4)⁷ × (3/4)⁰

≈ 0.0000

To calculate the variance of X, we need to calculate the expected value of X and the expected value of X squared:

E(X) = Σ k P(X = k)

= 0P(X=0) + 1P(X=1) + 2P(X=2) + 3P(X=3) + 4P(X=4) + 5P(X=5) + 6P(X=6) + 7P(X=7)

≈ 1.75

E(X²) = Σ k²P(X = k)

= 0²P(X=0) + 1²P(X=1) + 2²P(X=2) + 3²P(X=3) + 4²P(X=4) + 5²P(X=5) + 6²P(X=6) + 7²P(X=7)

≈ 4.56

Then, we can use the formula for the variance:

Var(X) = E(X²) - [E(X)]²

≈ 4.56 - (1.75)²

≈ 1.03

Rounding to 2 decimal places, the variance of X is 1.31.

To know more about Probability mass function visit:

https://brainly.com/question/30765833

#SPJ1

The complete question is as follows:

A small deck of four cards consists of one red card and three green cards. Draw 7 times with replacement. Assume each draw is a random selection of one card. Let X = the number of red cards drawn, compute the variance of X. Round to 2 decimal places.

Var(X) =

prove that the minimum value of the rayleigh quotient of a positive semi-definite, but not positive definite, operator is 0.

Answers

A positive semi-definite operator's rayleigh quotient must have a minimum value of zero to be considered positive.

Let A be a non-positive definite positive semi-definite operator. This proves that a non-zero vector x exists such that Ax = 0. The Rayleigh quotient of A with regard to x may thus be defined as follows:

[tex]R(x) = (x^T)Ax / (x^T)x[/tex]

A is positive semidefinite, hence for each vector x, (xT)Ax >= 0 is true. However, there is a non-zero vector x such that Ax = 0 if A is not a positive definite. In this instance, the Rayleigh quotient's numerator is 0, and as a result, the Rayleigh quotient is also 0. Since there is always a non-zero vector x such that Ax = 0, we may infer that the Rayleigh quotient's lowest value for a positive semi-definite but not positive definite operator is 0.

Read more about rayleigh quotient on:

https://brainly.com/question/20913723

#SPJ4

BRAINEST IF CORRECT! 25 POINTS.
What transformation of Figure 1 results in Figure 2?
Select from the drop-down menu to correctly complete the statement.
A ______ of Figure 1 results in Figure 2.

Answers

Answer:

its reflection

Step-by-step explanation:

a reflection is known as a flip. A reflection is a mirror image of the shape. An image will reflect through a line, known as the line of reflection. A figure is said to reflect the other figure, and then every point in a figure is equidistant from each corresponding point in another figure.

Answer:

It is Reflection. Check if it is in the list.

if 3 cos square root of 5 and ø€(180:360) .calculate without using a calculator A.2sin​

Answers

The measure of the angles obtained using trigonometric identities are;

sin(2·θ) = -(4·√5)/9cos(2·θ) = 1/9tan(2·θ) = -4·√5

What are trigonometric identities?

Trigonometric identities are mathematical equations that consists of the trigonometric functions and which are correct for the values of the angles entered into the equations.

The value of sin(2·θ) can be obtained by making use of the Pythagorean identity as follows;

cos²(θ) + sin²(θ) = 1

sin²(θ) = 1 - cos²(θ)

sin(θ) = √(1 - cos²(θ))

3·cos(θ) = √5

cos(θ) = √5/3

sin(θ) = √(1 - (√5/3)²) = 2/3

180° ≤ θ ≤ 360°, therefore, sin(θ) is negative, which indicates;

sin(θ) = -2/3

sin(2·θ) = 2·sin(θ)·cos(θ)

sin(2·θ) = 2×(-2/3) × (√5)/3 = -(4·√5)/9

sin(2·θ) = -(4·√5)/9

The double angle formula for cosines, indicates that we get;

cos(2·θ) = cos²(θ) - sin²(θ)

Therefore;

cos(2·θ) = ((√5)/3)² - (-2/3)² = 5/9 - 4/9 = 1/9

cos(2·θ) = 1/9

tan(2·θ) = sin(2·θ)/cos(2·θ)

Therefore;

tan(2·θ) = ((4·√5)/9)/(1/9) = 4·√5

tan(2·θ) = 4·√5

Learn more on trigonometric angle formula here: https://brainly.com/question/15353037

#SPJ1

Angela is riding on a circular Ferris wheel that has a 59-foot radius. After boarding the Ferris wheel, she traveled a distance of 44.3 feet along the arc before the Ferris wheel stopped for the next rider.

a) Make a drawing of the situation and illustrate relevant quantities.
b) The angle that Angela swept out along the arc had a measure of how many radians?
c) The angle that Angela swept out along the arc had a measure of how many degrees?

Answers

The motion of Angela, riding on the 59 feet radius Ferris wheel indicates;

a) Please find attached the drawing represent the situation created with MS Word

b) The angle Angela swept out along the ard is about 0.751 radians

c) The measure of the angle Angela swept out in degrees is about 43.02°

What is the radius of a circular figure?

The radius of a circular figure is the distance from the center of the figure to the circumference.

The specified parameters are;

Radius of the Ferris wheel = 59 feet

The distance along the arc, traveled by Angela, s = 44.3 feet

Let θ represent the angle Angela swept out along the arc, we get;

a) Please find attached the drawing of the situation created with MS Word

b)The formula for the arc length, s, of a circular motion is; s = r × θ

Where;

r = The radius of the circular motion, therefore;

θ = s/r

θ = 44.3/59 ≈ 0.751

The angle that Angela swept out, θ ≈ 0.751 radians

c) The angle swept out in degrees can be found as follows;

s = (θ/360) × 2 × π × r

Therefore;

44.3 = (θ/360) × 2 × π × 59

θ = 44.3° × 360°/(2 × π × 59) ≈ 43.02°

The angle Angela swept out is approximately 43.02°

Learn more on the motion of a Ferris wheel here:  https://brainly.com/question/11306671

#SPJ1

PLSS help mee with all 4 questions!!!

Answers

By answering the presented question, we may conclude that so by SAS congruency property we have said that all these triangles are similar.

What precisely is a triangle?

A triangle is a closed, a double geometric shape made up of three line segments known as sides that connect at three parameters known as vertices.

Triangles are differentiated by their angles and their sides. Triangles can be collinear (all sides equal), angles, or scalene dependent on their sides.

Triangles are classed as acute (all angles just under 90 degrees), right (one angle of approximately to 90 degrees), or ambiguous (all angles greater than 90 degrees).

The area of a triangle may be determined with the formula A = (1/2)bh, where A is the surface, b is really the right triangle base, and h is the triangle's height.

Here, from the figure, we have

Two sides of the triangle are equal.

And also each of the angles are also same.

Therefore, by SAS congruency property, we have say that all these triangles are similar.

Learn more about triangle here:

https://brainly.com/question/2773823

#SPJ1

The winning car in a race beat the second car by 19/100 of a second . The third car was 4/10 of a second behind the second car . By how much did the first car beat the third car ?

Answers

Add the times together:

19/100 + 4/10

Find the common denominator, which is 100 so rewrite 4/10 as 40/100

Now add:

19/100 + 40/100 = 59/100

The first car beat the third car by 59/100 seconds.

Please answer the following two questions

Answers

We obtain x 3 + 42 35 mod 49 by solving for x modulo 49. Thus, x ≡ 35 mod 56 is a solution to f(x) = 0 mod 56.

what is solution ?

A value or values of a set of variables that satisfy a formula or system of equations are referred to as solutions in mathematics. A remedy can also refers to a process of discovering such values.

given

(A) For x = 0, 1, 2, 3, 4, we calculate f(x) modulo 5:

Thus, f(x) = 0 mod 5 for x = 2, 3.

We compute the following to see if f'(x) is not congruent to 0 modulo 5 at either x = 2 or x = 3.

f'(x) = 2x

f'(2) = 4, f'(3) = 6

Thus, xo = 2 or xo = 3 will work.

(b) We use Hensel's lemma to lift solutions from mod 5 to mod 55 and mod 56.

For mod 5 to mod 55, we start with xo = 2. Since f'(2) = 4 is invertible modulo 5, we can find a unique solution modulo 25 using Hensel's lemma. We get:

f'(2) = 4

f(2) = 5

f(2) + 4(3)(x - 2) = 0 mod 25

f'(3) = 6

f(3) = 10

f(3) Plus 6(2)(x - 3) = 0 mod 49

We obtain x 3 + 42 35 mod 49 by solving for x modulo 49. Thus, x ≡ 35 mod 56 is a solution to f(x) = 0 mod 56.

To know more about solution visit :-

https://brainly.com/question/16989201

#SPJ1

The complete question is:-  Let f(x) = x2 + 1 € Z[X].

(a) Find an integer 0 < xo < 5 with f(x) = 0 mod 5 and f'(xo) # 0 mod 5.

(b) Use Hensel's lemma to find solutions to the congruences f(x) = 0 mod 55 and f(x) = 0 mod 56.

Suppose the current cost of gasoline is ​$2.93 per gallon. Find the current price index​ number, using the 1975 price of 56.7 cents as the reference value.

Answers

Answer:

Step-by-step explanation:

To find the current price index number using the 1975 price of 56.7 cents as the reference value, we can use the formula:

Price Index = (Current Price / Base Price) x 100

Where "Current Price" is the current cost of gasoline, and "Base Price" is the 1975 price of 56.7 cents.

Substituting the values given in the problem, we get:

Price Index = ($2.93 / $0.567) x 100

Price Index = 516.899

Therefore, the current price index number, using the 1975 price of 56.7 cents as the reference value, is 516.899.

if sin0<0 and cos>0, then the terminal point is determined by 0 is in:

Answers

the terminal point of the angle determined by sin(0) < 0 and cos(0) > 0 is in the fourth quadrant.

why it is and what is trigonometry?

If sin(0) < 0 and cos(0) > 0, then we know that the angle 0 is in the fourth quadrant of the unit circle.

In the unit circle, the x-coordinate represents cos(θ) and the y-coordinate represents sin(θ). Since cos(0) > 0, we know that the terminal point of the angle is to the right of the origin. And since sin(0) < 0, we know that the terminal point is below the x-axis.

The fourth quadrant is the only quadrant where the x-coordinate is positive and the y-coordinate is negative, so that is the quadrant where the terminal point of the angle lies.

Therefore, the terminal point of the angle determined by sin(0) < 0 and cos(0) > 0 is in the fourth quadrant.

Trigonometry is a branch of mathematics that deals with the relationships between the sides and angles of triangles. It focuses on the study of the functions of angles and their applications to triangles, including the measurement of angles, the calculation of lengths and areas of triangles, and the analysis of periodic phenomena.

To know more about Trigonometry visit:

https://brainly.com/question/29002217

#SPJ1

show that if x < y are real numbers then there are innitely many rational numbers b such that x < b < y. g

Answers

To show that there are infinitely many rational numbers between any two real numbers x and y, where x< y, we can use the Archimedean property of the real numbers.

The Archimedean property states that for any two positive real numbers a and b, there exists a positive integer n such that na>b. Let's choose a positive integer n such that 1/n< y-x. Then we can divide the interval(x,y) into n subintervals of equal length:

(x, y) = (x, x + (y - x)/n) ∪ (x + (y - x)/n, x + 2(y - x)/n) ∪ ... ∪ (x + (n - 1)(y - x)/n, y).

Each of these intervals  has length(y-x)/n, which is less than 1/n. therefore, there must be at least one integer k such that x+k(y-x) is a rational number. This is because the numerator k(y-x) is an integer, and the denominator n is a positive integer.Since there are n subintervals, we have found at least n different rational numbers between x and y.

However, since the choice of n was arbitrary we can choose a larger n to find even more rational numbers between x and y. Therefore, there must be infinitely many rational numbers between x and y.

To know more about Archimedean property refer to the link  brainly.com/question/14934545

#SPJ4

Let x and y be reals with x<y. Show that there are infinitely many rationals b such that x<b<y.

if you could please help i am having issues​

Answers

Since the p-value (0.0803) exceeds the significance threshold (0.05), the null hypothesis cannot be ruled out.

what is mean ?

The mean in mathematics is a measurement of a collection of numerical data's central tendency. It is determined by adding up all of the values in the set and dividing the result by the total number of values. This value is frequently referred to as the average value. The mean (or mathematical mean) is calculated as follows: (Sum of Values) / Mean (number of values)

given

The null hypothesis states that the mean number of units generated during the day and night shifts is the same. The contrary hypothesis (Ha) states that more units are created on average on the night shift than on the day shift.

"day" + "night"

Bravo! Night precedes day.

b. The following method can be used to calculate the test statistic:

t = sqrt(1/n night + 1/n day) * sqrt(x night - x day)

where s p is the pooled standard deviation and x night and x day are the sample averages, n night and n day are the sample sizes, and s p is represented by:

Sqrt(((n night - 1)*s night2 + (n day - 1)*s day2) / (n night + n day - 2)) yields the value s p.

S p is equal to sqrt(((74 - 1)*35 + (68 - 1)*28) / (74 + 68 - 2)), which equals 31.88.

t = (358 - 352) / (31.88 * sqrt(1/74 + 1/68)) = 1.19

1.19 is the test result.

the p-value is 0.0803 as a result (rounded to 4 decimal places).

Since the p-value (0.0803) exceeds the significance threshold (0.05), the null hypothesis cannot be ruled out.

To know more about mean visit:

https://brainly.com/question/30094057

#SPJ1

The complete question is :- Clark Heter is an industrial engineer at Lyons Products. He would like to determine whether there are more units produced on the night shift than on the day shift. The mean number of units produced by a sample of 68 day-shift workers was 352. The mean number of units produced by a sample of 74 night-shift workers was 358. Assume the population standard deviation of the number of units produced is 28 on the day shift and 35 on the night shift.

Using the 0.05 significance level, is the number of units produced on the night shift larger?

a. State the null and alternate hypotheses.

O : Day/Night: H:

Day Night

b. Compute the test statistic. (Negative values should be indicated by a minus sign. Round your answers to 2 decimal places.)

c. Compute the p-value. (Round your answer to 4 decimal places.) p-value

Math
rade> Y.9 Solve two-step equations: complete the solution GK7
2(p+ 4) = 12
P + 4 =
Social studies
Complete the process of solving the equation.
Fill in the missing term on each line. Simplify any fractions.
Р
Submit
Recommendations
Divide both sides by 2
Subtract 4 from both sides

Answers

P = 2 is the answer to the equation 2(p + 4) = 12.

Is it an equation or an expression?

An expression is made up of a number, a variable, or a combination of a number, a variable, and operation symbols. Two expressions are combined into one equation by using the equal symbol. For illustration: When you add 8 and 3, you get 11.

Divide the two among the terms between the parenthesis:

2p + 8 = 12

Add 8 to both sides of the equation, then subtract 8:

2p + 8 - 8 = 12 - 8

2p = 4

multiply both sides by two:

2p/2 = 4/2 \sp = 2

p = 2 is the answer to the equation 2(p + 4) = 12 as a result.

Simply put p = 2 back into the equation and simplify to obtain p + 4:

[tex]p + 4 = 2 + 4 = 6[/tex]

Hence, p + 4 = 6.

To know more about equation visit:-

https://brainly.com/question/29657983

#SPJ1

HELP! I WILL AMKE YOU BRAINLIEST BC THIS IS DUE TODAY!!!

Answers

Answer: 27.3

Step-by-step explanation:

I took the outcomes of the Aces from the trial and found the average and the answer I got was 27.3%

Hope this helps.

What is the simplest form of the radical expression?
show work please

Answers

let's recall that the conjugate of any expression is simply the same pair with a different sign between, so conjugate of "a + b" is just "a - b" and so on.  That said, let's use the conjugate of the denominator

[tex]\cfrac{\sqrt{2}+\sqrt{3}}{\sqrt{2}-\sqrt{3}}\cdot \cfrac{\sqrt{2}+\sqrt{3}}{\sqrt{2}+\sqrt{3}}\implies \cfrac{(\sqrt{2}+\sqrt{3})(\sqrt{2}+\sqrt{3})}{\underset{ \textit{difference of squares} }{(\sqrt{2}-\sqrt{3})(\sqrt{2}+\sqrt{3})}}\implies \cfrac{\stackrel{ F~O~I~L }{(\sqrt{2}+\sqrt{3})(\sqrt{2}+\sqrt{3})}}{(\sqrt{2})^2-(\sqrt{3})^2} \\\\\\ \cfrac{2+2\sqrt{2}\cdot \sqrt{3}+3}{2-3}\implies \cfrac{5+2\sqrt{6}}{-1}\implies \boxed{-5-2\sqrt{6}}[/tex]

The five-number summary of a data set is given below.

Minimum: 3 Q1: 12 Median: 15 Q3: 16 Maximum: 20

Which of the following equals 1.5(IQR)?

Answers

The required value is 1.5(IQR) equals 6.

What is Data set?

A dataset is a collection of facts that relates to a particular subject. The test results of each pupil in a particular class are an illustration of a dataset. Datasets can be expressed as a table, a collection of integers in a random sequence, or by enclosing them in curly brackets.

According to question:

The IQR (interquartile range) is the difference between the third quartile (Q3) and the first quartile (Q1). So, we first need to calculate IQR:

IQR = Q3 - Q1 = 16 - 12 = 4

Now we can calculate 1.5 times the IQR:

1.5(IQR) = 1.5(4) = 6

Therefore, 1.5(IQR) equals 6.

To know more about Data set visit:

brainly.com/question/14893265

#SPJ1

Complete question:

The five-number summary of a data set is given below.

Minimum: 3 Q1: 12 Median: 15 Q3: 16 Maximum: 20

Which of the following equals 1.5(IQR)?

after completing your data analysis, the write-up should include a discussion of which of the following?

Answers

After completing your data analysis, the write-up should only include a discussion of the steps of the IMPACT model that really matter.

Data analysis is the methodical application of logical and/or statistical approaches to explain and demonstrate, summarise and assess, and assess data. Different analytical techniques "offer a mechanism of deriving inductive inferences from data and differentiating the signal (the phenomena of interest) from the noise (statistical fluctuations) inherent in the data," according to Shamoo and Resnik (2003).

The proper and accurate interpretation of study findings is a crucial part of preserving data integrity. Inadequate statistical analyses distort scientific results, confuse lay readers, and may have a detrimental impact on how the general public views research (Shepard, 2002). Integrity concerns apply equally to the study of non-statistical data.

Impact analysis examines required data to determine the advantages and disadvantages of any change. Even in a well evolved system, adjustments are inevitable as the world develops. Modifications might occur for a number of reasons, including modifications to company demands, changes in customer requirements, or the introduction of new technology.

Learn more about Data Analysis:

https://brainly.com/question/20714266

#SPJ4

Examine the following graphed systems of linear inequalities. Select the points below that are solutions to each system of inequalities. Select TWO that apply.
1. 2.
(2,3) (0,0)
(4,3) (4,3)
(-7,6) (6,1)
(-2,3) (2-5)
I need help D: pls ​

Answers

The solution of the graphs are as follows

first graph

(2, 3)(4, 3)

second graph

(4, 3)(6, 1)

How to find the ordered pair that are solution of the graph

The graphs consist of two sets of equations plotted, each has shade peculiar to the equation.

The solution of the graph consist of the ordered pair that fall within the parts covered by the two shades

For the first graph by the left, the solutions are

(2, 3)(4, 3)

For the second graph by the left, the solutions are

(4, 3)(6, 1)

Learn more about inequality graphs at:

https://brainly.com/question/24372553

#SPJ1

You need 2 jugs of orange juice for every 3 batches of punch you make. How many jugs of orange juice do you need if you make 24 batches of punch? 16​

Answers

Answer: 16 Jugs of orange juice

Step-by-step explanation:

Let

J = Jugs of orange juice

P = Batches of punch

2J = 3P

Therefore to find what 1 P equals divide both sides by 3 giving:

2/3 J = 1P

Using this ratio, take it and apply it to the given question:

2/3 J = 1P

therefore:

24 x 2/3 = Needed J

= 16J

Assume each newborn baby has a probability of approximately 0.49 of being female and 0.51 of being male. For a family with four children, let X = number of children who are girls.Find the probability that the family has two girls and two boys. (Round to four decimal places as needed.)

Answers

The probability that a family with four children has two girls and two boys is 0.3734, or approximately 0.3734 rounded to four decimal places. We can solve it in the following manner.

The gender of each child is independent of the gender of their siblings, and can be modeled as a Bernoulli random variable with parameter 0.49 for female and 0.51 for male. Since we are interested in the number of girls in a family of four children, X follows a binomial distribution with n = 4 and p = 0.49.

The probability of having exactly 2 girls and 2 boys can be calculated using the binomial probability mass function:

P(X = 2) = (4 choose 2) * 0.49² * 0.51²

= 6 * 0.2401 * 0.2601

= 0.3734

Therefore, the probability that a family with four children has two girls and two boys is 0.3734, or approximately 0.3734 rounded to four decimal places.

Learn more about probability here brainly.com/question/11234923

#SPJ4

Rehan has $50 in his wallet. The money he has left over after buying two boxes of cookies is given by the equation 50- x = 38, where x represents the cost of two boxes of cookies. What is the cost of two boxes of cookies, in dollars?​

Answers

Answer:

We are given the equation 50 - x = 38, where x represents the cost of two boxes of cookies.

To find the cost of two boxes of cookies, we need to isolate the variable x.

First, we will subtract 38 from both sides of the equation:

50 - x - 38 = 0

Simplifying:

12 - x = 0

Now, we will add x to both sides of the equation:

12 = x

Therefore, the cost of two boxes of cookies is $12.

The roots of a quadratic equation a x +b x +c =0 are (2+i √2)/3 and (2−i √2)/3 . Find the values of b and c if a = −1.

Answers

[tex]\begin{cases} x=\frac{2+i\sqrt{2}}{3}\implies 3x=2+i\sqrt{2}\implies 3x-2-i\sqrt{2}=0\\\\ x=\frac{2-i\sqrt{2}}{3}\implies 3x=2-i\sqrt{2}\implies 3x-2+i\sqrt{2}=0 \end{cases} \\\\\\ \stackrel{ \textit{original polynomial} }{a(3x-2-i\sqrt{2})(3x-2+i\sqrt{2})=\stackrel{ 0 }{y}} \\\\[-0.35em] ~\dotfill[/tex]

[tex]\stackrel{ \textit{difference of squares} }{[(3x-2)-(i\sqrt{2})][(3x-2)+(i\sqrt{2})]}\implies (3x-2)^2-(i\sqrt{2})^2 \\\\\\ (9x^2-12x+4)-(2i^2)\implies 9x^2-12x+4-(2(-1)) \\\\\\ 9x^2-12x+4+2\implies 9x^2-12x+6 \\\\[-0.35em] ~\dotfill\\\\ a(9x^2-12x+6)=y\hspace{5em}\stackrel{\textit{now let's make}}{a=-\frac{1}{9}} \\\\\\ -\cfrac{1}{9}(9x^2-12x+6)=y\implies \boxed{-x^2+\cfrac{4}{3}x-\cfrac{2}{3}=y}[/tex]

Select the correct answer.
Simplify the following expression.

Answers

Answer:

a

Step-by-step explanation:

889-0494444)()54837

Other Questions
what do you think are the effects of selective judgment among other religions?( i need 8-10 sentences and its for islam) select the following true statements about memory, in accordance with the information in this module's videos. Assume that the readings at freezing on a batch of thermometers are normally distributed with a mean of 0C and a standard deviation of 1.00C. A single thermometer is randomly selected and tested. Find the probability of obtaining a reading between 0.59C and 0.88C. Give an example to show that the Monotone Convergence Theorem (3.11) can fail if the hypothesis that f1, f2, ... are nonnegative functions is dropped. 3.11 Monotone Convergence Theorem Suppose (X, S, u) is a measure space and 0 < fi < f2 10 POINTS!!!NEED HELP ASAP PLEASE HELP FIND THE AREA AND THE PERIMETER!! The pH in the intermembrane space of the mitochondria should be_____ compared to the matrix due to theA.higher; higher concentration of protons in the intermembrane space B.higher; lower concentration of protons in the intermembrane space C.lower; higher concentration of protons in the intermembrane spaceD.lower; lower concentration of protons in the intermembrane space Supporting individuality is providing forsomeone's personaland individual- (a) Compute the specific heat at constant volume of nitrogen (N2) gas, and compare it with the specific heat of liquid water. The molar mass of N2 is 28.0 g/mol. (b) You warm 1.00 kg of water at a constant volume of 1.00 L from 20.0C to 30.0C in a kettle. For the same amount of heat, how many kilograms of 20.0C air would you be able to warm to 30.0C? What volume (in liters) would this air occupy at 20.0C and a pressure of 1.00 atm? Make the simplifying assumption that air is 100% N2. Write a paragraph that evaluates Odysseuss qualifications as an epic hero.????? HELP BRO HELO Amy runs each lap in 4 minutes. She will run less than 7 laps today. What are the possible numbers of minutes she will run today? Triangle lmn will be dilated with respect to the origin by a scale factor of 1/2what are the new coordinates of LMN a 3-ary tree is the tree in which every internal node has exactly 3 children. how many leaf nodes are there in a 3-are tree with 6 internal nodes the formation of anti slavery organizations during the 1830s and 1840s was an example of interest groups that formed as a result of What would you do if you had a magic pencil? How would you used it And why? Explain pls Why did the number of male voters increase during the 1824 presidential election?A. Many territories allowed formerly enslaved persons to vote.B. The federal government required males over a certain age to vote.C. Many states loosened or eliminated property requirements for voting.D. The federal government promised land grants to men who voted. FOR 50 POINTS AND BRAINLIEST!! PLEASE HELP ASAP! thank you :) When price and quantity move in the opposite direction, it is the law of__________________________.(U.S. Government and Economics) There are several elements to internal controls. Which of the following would not address the issue of having cash transactions reported in the accounting records?A. One employee would have access to the cash register.B. The cash drawer should be closed out, and cash and the sales register should be reconciled on a prenumbered form.C. Ask customers to report to a manager if they do not receive a sales receipt or invoice.D. The person behind the cash register should also be responsible for making price adjustments. what is the digestive enzyme that begins the process of converting starches in the food into a disaccharide called maltose? the democratic party group of answer choices favored strict limitations on voting rights. was organized to oppose andrew jackson. was similar in policy and constituencies to the federalist party. supported both agrarian democratic principles and the continuation of southern slavery. was almost destroyed by the election of 1828